\documentclass[mode = exam, twocolumn]{hduthesis} \usepackage[fontset = fandol]{ctex} \usepackage[warnings-off={mathtools-colon,mathtools-overbracket}]{unicode-math} \usepackage{libertine} \title{\bfseries 24Fall \textsc{Hdu} 「模拟电子技术」期末试题} \author{} \date{} \ctikzset { amplifiers/fill = cyan!20, resistors/fill = violet!20, sources/fill = yellow!20, diodes/fill = red!20, resistors/scale = .6, capacitors/scale = .6, diodes/scale = .6, sources/scale = .8, inductors/scale = .6, } \tikzset {every node/.style = {font = \small}} \begin{document} \maketitle \begin{problem}[12 pt] 如图所示为加减运算电路,求输出电压 $v_o$ 的表达式. \centering \begin{circuitikz} \draw (0,0) node [ right ] {$v_o$} to [ short, o- ] ++ (-1,0) node [ op amp, anchor = out ] (A) {$\mathrm A$}; \draw (-6.5,2) node [ left ] {$v_{i_1}$} coordinate (vi1) to [ short, o-, european resistor, l = {$R_1 = \qty{30}\kohm$}] ++ (2.5,0) coordinate (R1) to [ european resistor, l = {$R_3 = \qty{60}\kohm$} ] (R1 -| A.out) to [ short, -* ] (A.out); \draw (A.- -| vi1) node [ left ] {$v_{i_2}$} to [ short, o-, european resistor, l = ${R_2 = \qty{60}\kohm}$ ] ++ (2.5,0) coordinate (R2) -- (A.-); \draw (A.+ -| vi1) node [ left ] {$v_{i_3}$} to [ short, o-, european resistor, l = ${R_4 = \qty{20}\kohm}$ ] ++ (2.5,0) coordinate (R4) -- (A.+); \draw (R1) to [ short, *-* ] (R2) node [ above right ] {$v_n$} (R4) node [ below right ] {$v_p$} to [ short, *- ] ++ (0,-.5) to [ european resistor, l_ = $R_5$ ] ++ (0,-1) node [ rground ] {}; \end{circuitikz} \end{problem} \begin{solution} \end{solution} \begin{problem}[12 pt] 如图所示放大电路 \begin{enumerate} \item 请判断反馈放大电路的反馈组态. \item 在深度负反馈条件下,求反馈系数和闭环增益表达式. \end{enumerate} \centering \begin{circuitikz} \draw (0,0) node (ground) [ rground ] {} to [ european resistor, l = $R_1$] ++ (0,1.25) --++ (0,.25) node [ anchor = S, nigfete, solderdot ] (fet) {$T_1$} (fet.G) to [ short, -o ] ++ (-2,0) node [ left ] (vi) {$v_i$}; \draw ([xshift = -1cm]fet.G) to [ short, *-, european resistor, l_ = $R_g$ ] ++ (0,-2) coordinate (Rg) to (Rg |- ground) node [ rground ] {}; \draw (fet.S) to [ short, *-, european resistor, l_ = $R_2$ ] ++ (2,0) to [ short, -o ] ++ (.5,0) node [ right ] (vo) {$v_o$}; \draw (fet.D) to [ european resistor, l = $R_d$ ] ++ (0,2) to [ short, -o ] ++ (2.5,0) node [ right ] (VDD) {$V_\text{DD}$}; \draw (fet.D) to [ short, *- ] ++ (1,0) node [ pnp, anchor = B ] (pnp) {$T_2$}; \draw (pnp.E) to [ european resistor, l = $R_e$ ] ++ (0,1) coordinate (Re) (Re) to [ short, -* ] (VDD -| Re); \draw (pnp.C) to [ short, -* ] (vo -| pnp.C); \end{circuitikz} \end{problem} \begin{solution} \end{solution} \newpage \begin{problem}[12 pt] 如图电路,D 为硅二极管,$V_\text{DD} = \qty2\V$,$R = \qty1\kohm$,正弦信号 $v_s = 100 \sin(2\pi \times 50t) \unit\mV$,求输出电压 $v_o$. \centering \begin{circuitikz}[american] \draw (0,0) node [ below ] {$+$} to [ short, o- ] ++ (-1.5,0) to [ stroke diode, invert, l_ = D ] ++ (-3,0) to [ european voltage source, l_ = $v_s$ ] ++ (0,-2) to [ battery1, l_ = $V_\text{DD}$] ++ (0,-1) to [ short, -o ] ++ (4.5,0) node [ above ] {$-$}; \draw (-1.5,0) to [ short, *-*, european resistor, l_ = $R$ ] ++ (0,-3); \node at (0,-1.5) {$v_o$}; \end{circuitikz} \end{problem} \begin{solution} \end{solution} \begin{problem}[15 pt] 电路如下图所示,硅 BJT 三极管的 $\beta = 100$,$C_1$ 和 $C_2$ 为隔直耦合电容. \begin{enumerate} \item 发射极静态电流 $I_\text{BQ} = \qty1\mA$,求 $R_e$ 的值. \item 集电极电压 $V_\text{CQ} = \qty5\V$,求 $R_c$ 的值. \item $R_L = \qty5\kohm$,求电压增益 $A_\text{VB}$. \end{enumerate} \centering \begin{circuitikz} \draw (0,0) node [ npn, anchor = B ] (npn) {$T$} to [ european resistor, l_ = {$R_b = 500$} ] ++ (-2,0) to [ european voltage source, l_ = $V_s$ ] ++ (0,-2) coordinate (ground) node [ rground ] {}; \draw (npn.C) to [ european resistor, l_ = $R_c$] ++ (0,1.5) node [ vcc ] {$+\qty{15}\V$} (npn.E) to [ european resistor, l = $R_e$] ++ (0,-1.5) node [ vee ] {$-\qty{15}\V$}; \draw (npn.E) to [ short, *-, C = $C_2$ ] ++ (1.75,0) coordinate (C2) (C2 |- ground) node [ rground ] {} -- (C2); \draw (npn.C) to [ short, *-, C = $C_1$ ] ++ (2.5,0) node [ below left ] {$+$} to [ european resistor, l = $R_L$] ++ (0,-1.75) coordinate (RL) node [ above left ] {$V_0$}(RL |- ground) node [ rground ] {} node [ left ] {$-$} -- (RL); \end{circuitikz} \end{problem} \begin{solution} \end{solution} \newpage \begin{problem}[15 pt] 电路如图所示,设 MOSFET 的参数为 $V_\text{TN} = \qty1\V$, $K_n = \qty{0.8}{\mA/\V^2}$,$\lambda = 0$. \begin{enumerate} \item 试判断场效应管类型,及其工作区. \item 求静态工作点. \item 画出电路的微变等效电路,求电路的电压增益 $A_v$,输入电阻 $R_i$ 和输出电阻 $R_o$. \end{enumerate} \centering \begin{circuitikz} \draw (0,0) node [ below ] {$+$} to [ short, o-, C = $C_1$ ] ++ (1.75,0) --++ (.75,0) node (fet) [ anchor = G, nigfete ] {$T$} (fet.S) --++ (0,-2) node [ rground ] (ground) {}; \draw (fet.D) to [ european resistor, l_ = {$R_d = \qty{3}\kohm$} ] ++ (0,2) node [ vdd ] (vdd) {$+\qty5\V$}; \draw (0,-2) node [ above ] {$-$} coordinate (negative) to [ short, o-* ] (negative -| ground); \draw (1.75,-2) to [ short, *-*, european resistor, l_ = ${R_{g_2} = \qty{20}\kohm}$ ] (1.75,0) to [ european resistor, l_ = ${R_{g_1} = \qty{20}\kohm}$ ] ++ (0,2.5) --++ (0,.5) coordinate (Rg1) to [ short, -* ] (Rg1 -| vdd); \draw (fet.D) to [ short, *-o, C = $C_2$ ] ++ (2.5,0) node [ below ] {$v_o$}; \end{circuitikz} \end{problem} \begin{solution} \end{solution} \begin{problem}[12 pt] 双电源互补对称电路如图所示,设 $V_\text{CC} = \qty{12}\V$,$R_L = \qty{12}\ohm$, $v_i$ 为正弦波,求 \begin{enumerate} \item 忽略 BJT 的饱和压降,负载上可能得到的最大输出功率 $P_\text{om}$. \item 直流电源供给的功率 $P_v = \frac2\pi \cdot \frac{V_\text{CC}}{R_L}$, 求放大器的效率 $\eta$. \end{enumerate} \centering \begin{circuitikz} \draw (0,0) node [ vee ] {$-V_\text{CC}$} --++ (0,.25) node [ pnp, anchor = C ] (pnp) {$T_2$} (pnp.E) node [ npn, anchor = E ] (npn) {$T_1$} (npn.C) --++ (0,.25) node [ vcc ] {$+V_\text{CC}$}; \draw (pnp.E) to [ short, *-o ] ++ (.9,0) --++ (.6,0) node [ right ] {$+$} to [ european resistor, l_ = $R_L$ ] ++ (0,-1.5) node [ right ] {$-$} node [ rground ] {}; \draw ([xshift = -1.5cm]pnp.E) coordinate (vi) to [ short, *-o ] ([xshift = -2.5cm]pnp.E) node [ left ] {$v_i$}; \draw (npn.B) -| (vi) |- (pnp.B); \end{circuitikz} \end{problem} \begin{solution} \end{solution} \newpage \begin{problem}[14 pt] 已知如图所示电路,设硅三极管 BJT 的 $\beta = 100$. \begin{enumerate} \item 计算电路的静态工作点. \item 计算双端输入、双端输出时的差模电压增益. \end{enumerate} \centering \begin{circuitikz} \draw (0,0) node [ vee ] {VEE $-\qty6\V$} to [ european resistor, l_ = {$R_e = \qty{5.3}\kohm$} ] ++ (0,1) to [ short, -* ] ++ (0,.25) coordinate (base); \draw (base) -| (-1,2) node (npn1) [ npn, anchor = E ] {$T_1$} (npn1.B) to [ short, -o ] ++ (-.25,0) node [ left ] {$V_{i_1}$} (npn1.C) to [ european resistor, l = {$R_{e_1} = \qty{6.2}\kohm$} ] ++ (0,2) --++ (1,0) coordinate (top); \draw (base) -| (1,2) node (npn2) [ npn, xscale = -1, anchor = E, reversed ] {\ctikzflipx{$T_2$}} (npn2.B) to [ short, -o ] ++ (.25,0) node [ right ] {$V_{i_2}$} (npn2.C) to [ european resistor, l_ = {$R_{e_2} = \qty{6.2}\kohm$} ] ++ (0,2) to [ short, -* ] ++ (-1,0); \draw (top) --++ (0,.25) node [ vcc ] {VCC $+\qty{6}\V$}; \draw (npn1.C) to [ short, *-o ] ++ (.25,0) node [ right ] {$V_{e_1}$} (npn2.C) to [ short, *-o ] ++ (-.25,0) node [ left ] {$V_{e_2}$}; \end{circuitikz} \end{problem} \begin{solution} \end{solution} \begin{problem}[8 pt] 电路如图所示,试用相位平衡条件判断能不能振荡. 如果能,请说明理由; 如果不能,也请说明理由,并最少程度上修改电路使其能产生振荡. \centering \begin{circuitikz}[american] \draw (0,0) to [ short, *-] ++ (.25,0) node [ npn, anchor = B ] (npn) {$T$} node [ transformer, anchor = A2 ] (trans) at (npn.C) {$L$} node [ circ ] at (npn.C) {} node [ circ ] at (trans.outer dot A2) {} node [ circ ] at (trans.outer dot B1) {} node [ rground ] at (trans.B2) {}; \draw (trans.A1) --++ (-.5,0) to [ C = $C$ ] ([xshift = -.5cm]trans.A2) -- (trans.A2); \draw (npn.B) to [ C = $C_b$] ++ (-1.5,0) |- (3,-3) |- ([xshift = .25cm]trans.B1) -- (trans.B1); \draw (trans.A1) --++ (0,.5) coordinate (vcc) to [ short, -o ] ++ (-2,0) node [ below ] {$V_\text{CC}$}; \draw (npn.E) to [ short, *-*, european resistor, l_ = $R_E$ ] ++ (0,-1.5) node [ rground ] (ground) {}; \draw (npn.E) --++ (1,0) coordinate (Ce) to [ C = $C_e$ ] (Ce |- ground) -- (ground) to (ground -| npn.B) --++ (-.25,0) to [ short, -*, european resistor, l = $R_{b_2}$] ([xshift = -.25cm]npn.B) coordinate (Rb2) to [ short, -*, european resistor, l = $R_{b_1}$] (Rb2 |- vcc); \end{circuitikz} \end{problem} \end{document}